Pharm Exam 4 Practice Questions

¡Supera tus tareas y exámenes ahora con Quizwiz!

Which of the following patient has less risk associated with HRT? A- 45F with family history of osteoporosis with vasomotor symptoms. B- 55F with smoker, DM, HTN with vasomotor symptoms. C- 62F with CAD with insomnia. D- 40F with family history of endometrial cancer.

A- 45F with family history of osteoporosis with vasomotor symptoms.

Which of the following patients should not be prescribed COC? A- 22 y/o healthy, with a history of chlamydia. B- 40 y/o tobacco user. C- 30 y/o healthy female. D- 24 y/o with history of DVT E- 45 y/o healthy, s/p hysterectomy

B - 40 y/o tobacco users & D- 24 y/o with history of DVT

Which of the following is an indication for treatment with anabolic androgens? A- Hypotension B- AIDS wasting C- Improving sports performance D- Prostate cancer E- Anemia

B - AIDS wasting

Which of the following is the longest acting? A- Sildenafil B- Tadalafil C- Vardenafil

B - Tadalafil

A 52-year old woman presents with no menses for two months, hot flashes, vaginal dryness, and mood swings. She has no significant past medical history. She has normal vital signs. She has normal labs including thyroid and pituitary function. FSH level is elevated. An endometrial biopsy reveals no evidence of endometrial hyperplasia. She has no history of cancer or prior history of abnormal vaginal bleeding. How would your counsel her regarding the use of hormone replacement therapy. A- Progestin therapy alone because she still has a uterus. B- Estrogen and progestin combination therapy. C- HRT will increase her risk of osteoporosis. D- HRT will decrease her risk of breast cancer. E- HRT will decrease her risk of cardiovascular disease.

B- Estrogen and progestin combination therapy.

Which of the following drugs acts as an agonist at the GnRH receptor? A- Tadalafil B- Leuprolide C- Ulipristal D- Ospemifene E- Mifepristone

B- Leuprolide

What is a contraindication for flutamide use? A- Hypotension B- Nitrates C- Hepatic impairment D- History of DVT/PE E- Prostate cancer

c - Hepatic impairment

Name 4 progestogens/progestins.

•Desogestrel •Drospirenone •Levonorgestrel •Medroxyprogesterone •Morethindrone •Norethindrone acetate •Norgestimate •Norgestrel •Progesterone

Name 3 estrogens.

•Estradiol •Estrone •Estriol •Ethinyl estradiol

What is a potential side effect/risk of tamoxifen? A- Endometrial cancer B- Breast cancer C- Osteoporosis D- Ovarian cancer

A - Endometrial cancer

Mr. X is prescribed leuprolide for treatment of testicular cancer. Which of the following is a potential side effect of leuprolide? A- Headache, gynecomastia and hot flashes B- Hypotension C- Acne D- Endometrial hyperplasia E- Hypothyroidism

A - Headache, gynecomastia and hot flashes

What is a contraindication for the use of estrogen-containing oral contraceptives? A- History of migraines with aura B- Current smoking C- Current hypertension D- History of chlamydia E- History of PCOS

A - History of migraines with aura Other options aren't a true contraindication but not preferred

Which of the following drugs can be associated with reflex tachycardia, ejaculatory dysfunction and rarely, intraoperative floppy iris syndrome during cataract surgery. A- Tamsulosin B- Sildenafil C- Finasteride D- Mifepristone E- Stanozolol

A - Tamsulosin

Which of the following are less likely to be associated with orthostatic hypotension? A- Tamsulosin B- Terazosin C- Alfuzosin D- Doxazosin E- Prazosin ****Tamsulosin and alfuzosin are more uroselective.

A - Tamsulosin & C- Alfuzosin both are are more uroselective.

Mrs. Black is a 46-year-old female patient who is having frequent hot flashes, insomnia and vaginal atrophy. She presents in the office today asking what she can do to help her symptoms. Her past medical history is significant for hypertension, GERD and diabetes. Her diabetes is well controlled with metformin, diet and exercise. She had a hysterectomy at the age of 43 for heavy menstrual bleeding secondary to fibroids. Which of the following are treatment options of her menopausal symptoms? A- Transdermal estrogen B- Combination oral contraception C- Levonorgestrel D- Medroxyprogesterone acetate E- Mifepristone

A - Transdermal estrogen; could take COC because she had a hysterectomy

Which of the following are potential side effects of raloxifene? A- Vaginal bleeding B- Hypotension C- Decrease HDL D- Osteoporosis E- Worsening heart disease

A - Vaginal bleeding; used for osteoporosis and has an effect on the endometrium

Which of the following is/are contraindication(s) for HRT? Select all that apply A- History of breast cancer B- Liver disease C- Osteoporosis D- Coronary artery disease E- Hysterectomy

A,B & C History of breast cancer Liver disease CAD

Alprostadil is administered locally. Which of the following is CORRECT regarding local administration of alprostadil? A. Local administration of alprostadil allows for low systemic absorption. B. Local administration of alprostadil increases the chance of drug interactions. C. Local administration of alprostadil is accomplished by way of a cream. D. Local administration of alprostadil causes changes in color vision.

A. Local administration of alprosta- dil allows for minimal systemic absorption. This makes alprostadil associated with few drug interactions. Alprostadil is administered by injection or urethral suppository, not a cream. Because there is little systemic absorption, and alprostadil does not affect PDE-6, changes in color vision are not likely.

Which of the following is a unique property of SERMs? A- Act as agonists in some tissues and antagonists in other tissues B- Activate a unique plasma membrane-bound receptor C- Have both estrogenic and progestational agonist activity D- Inhibit the aromatase enzyme required for estrogen synthesis E- Produce estrogenic effects without binding to estrogen receptors

A: SERMs such as tamoxifen and raloxifene exhibit tissue-specific estrogenic and antiestrogenic effects

What drug is an antagonist of progesterone receptors? A- Raloxifene B- Mifepristone C- Clomiphene D- Ulipristal E- Misoprostol

ANSWER = B & D A- SERM - antagonist in breast and endometrium, agonist at bone B- Blocks progesterone receptor C- SERM - antagonist at hypothalamus D- Selective progesterone receptor modulator - Binds to progesterone receptor E- Prostaglandin analog

What are potential side effects of alfuzosin? Select all that apply. A- Reflex tachycardia B- Ejaculatory dysfunction C- Headache D- Orthostatic hypotension E- Dizziness

All are correct !!!

Which of the following is CORRECT regarding finasteride? A. Finasteride is associated with significant hypotension. B. Finasteride is associated with birth defects. C. Finasteride is effective within 2 weeks of initiation. D. Finasteride is renally eliminated.

B. Because finasteride inhibits the con- version of testosterone to its active form, it may cause significant developmental defects in the male genitalia of a developing fetus. As such, it is contraindicated in preg- nancy. Unlike the α-blockers, the 5-α reductase inhibitors are not associated with hypotension. Finasteride may take up to 12 months before it is effective. Finally, finasteride is metabolized via CYP450 and is not renally eliminated

Which of the following statements is CORRECT regard- ing the mechanism of action of phosphodiesterase-5 (PDE-5) inhibitors? A. PDE-5 inhibitors increase prostaglandin production. B. PDE-5 inhibitors enhance the effect of nitric oxide. C. PDE-5 inhibitors cause vasoconstriction of the erection chamber. D. PDE-5 inhibitors antagonize cyclic GMP.

B. PDE-5 inhibitors enhance the effect of nitric oxide by preventing the breakdown of cGMP. PDE-5 inhibitors do not affect prostaglandin production. Although blood is drawn to the erection chamber, PDE-5 inhibitors allow for this via vasodilation, not vasoconstriction. PDE-5 inhibitors prevent the breakdown of cGMP but do not antag- onize its action.

Which of the following is an important difference between terazosin and tamsulosin? A. Terazosin blocks α1A receptors, while tamsulosin blocks α1A and α1B receptors. B. Terazosin blocks α1A and α1B receptors, while tamsulosin blocks α1A receptors. C. Terazosin blocks 5-α reductase, while tamsulosin blocks PDE-5. D. Terazosin must be taken with food, while tamsulosin can be taken on an empty stomach.

B. Tamsulosin is more selective for the α1A receptor, found in the prostate. Terazosin blocks α1A; however, terazosin also blocks α1B. Neither one blocks 5-α reductase nor PDE-5. Tamsulosin should be taken with food, while terazosin does not need to be taken with food.

Which of the following is correct regarding α-adrenergic blockers? A. α-Adrenergic blockers are used in the treatment of hypotension in anaphylactic shock. B. α-Adrenergic blockers are used in the treatment of benign prostatic hyperplasia (BPH). C. α-Adrenergic blockers may cause bradycardia. D. α-Adrenergic blockers are used in the treatment of asthma. E. α-Adrenergic blockers reduce the frequency of Urination.

B. α-Adrenergic blockers are used in the treatment of BPH because of their relaxant effect on prostate smooth muscles. Being antihypertensive agents, they are not useful in treating hypotension in anaphylaxis. α-Adrenergic blockers generally cause reflex tachycardia (not bradycardia) due to the significant drop in blood pres- sure caused by them. α-Adrenergic blockers have no sig- nificant effects on bronchial tissues and are not useful in treating asthma. They increase (not reduce) the frequency of urination by relaxing the internal sphincter of the urinary bladder, which is controlled by α1 receptors.

A young woman complains of abdominal pain at the time of menstruation. Careful evaluation indicates the presence of significant endometrial deposits on the pelvic peritoneum. Which of the following is the most appropriate medical therapy for this patient? A- Flutamide, orally B- Medroxyprogesterone acetate by intramuscular injection C- Norgestrel as an IUD D- Oxandrolone by intramuscular injection E- Raloxifene orally

B: In endometriosis, suppression of ovarian function and production of gonadal steroids are useful. Intramuscular injection of relatively large doses of medroxyprogesterone provides 3 months of an ovarian suppressive effect because of inhibition of pituitary production of gonadotropins

Men who use large doses of anabolic steroids are at increased risk of which of the following? A- Anemia B- Cholestatic jaundice and elevation of aspartate transaminase levels in the blood C- Hirsutism D- Hyperprolactinemia E- Testicular enlargement

B: In men, large doses of anabolic steroids are associated with liver impairment, including cholestasis and elevation of serum concentrations of transaminases.

Mr. T is a 65 y/o M presenting with urinary urgency. After some testing in the office and a good history and physical, you suspect his symptoms are secondary to BPH. Which of the following is a first-line treatment for BPH? A- Vardenafil B- Tolterodine C- Alfuzosin D- Finasteride E- Flutamide

C- Alfuzosin

What is the mechanism of action of alfuzosin? A- Prolonged use causes LH pituitary receptors to downregulate, LH levels are decreased and ultimately, testosterone production is significantly reduced B- Results in increased levels of cGMP which promotes smooth muscle relaxation. C- Blocks the alpha-1 adrenergic receptors in the periphery which results in smooth muscle relaxation. D- Competes with natural hormone for binding to the androgen receptor.

C- Blocks the alpha-1 adrenergic receptors in the periphery which results in smooth muscle relaxation.

Which of the following increases the chance of twins or triplet pregnancies? A- Raloxifene B- Flutamide C- Clomiphene D- Mifepristone E- Levonorgestrel

C- Clomiphene

The onset of ovarian function may be due to maturation of brain centers that withdraw an inhibition of cells in the hypothalamus allowing them to produce: A- FSH B- LH C- GnRH D- Estrogen E- TSH

C- GnRH

35-year old female presents for routine follow-up at 7 weeks gestation. Ultrasound reveals multiple anomalies incompatible with life. You discuss options for termination of pregnancy. Which of the following can be used for termination of pregnancy? A- Tamoxifen B- Estrogen C- Mifepristone D- Copper IUD E- Levonorgestrel

C- Mifepristone

A 35-year-old female comes in to discuss a new birth control method. She is two months postpartum and is no longer breast-feeding. She used combination oral contraceptives in the past but she now has frequent migraine headaches. She also has trouble remembering taking the pills. Her past history is notable for depression and obesity. She has regular but at times heavy period. She often has premenstrual cramping and NSAIDs are an effective relief. She denies history of STDs. She would like to have one more child but not for a couple of years. She smokes 1 pack of cigarettes daily. What contraceptive methods would be best for this patient? A- Progesterone only pill B- NuvaRing (ethinyl estradiol and etonogestrel) C- Mirena IUD (levonorgestrel IUD) D- Barrier methods E- Ortho Evra patch (COC)

C- Mirena IUD (levonorgestrel IUD)

Which hormone is dominant during the luteal phase of the menstrual cycle? A- Estrogen B- GnRH C- Progesterone D- LH E- Testosterone

C- Progesterone

Which of the following is considered first-line treatment for BPH? A- Tadalafil B- Oxybutynin C- Tamsulosin D- Finasteride E- Flutamide

C- Tamsulosin

To provide blockage of estrogen in premenopausal women, leuprolide and the goserelin are sometimes given to prevent the need for oophorectomy in patients with certain types of breast cancer. How to they work? A- They block LH and FSH on the ovary. B- They decreased level of adrenal androgens. C- They decrease the release of LH and FSH from the pituitary. D- They bind to estrogen receptors.

C- They decrease the release of LH and FSH from the pituitary.

Which of the following is mediated by progestogens? A- Stimulates endometrial growth. B- Rising levels will trigger LH surge. C- Thickens cervical mucus and impairs progress of egg in fallopian tube. D- Breast pain and bloating. E- Increased risk of thromboembolic disorder in some patients.

C- Thickens cervical mucus and impairs progress of egg in fallopian tube.

Mr. Herbert is a 29-year old female to male transgender patient. He is currently taking testosterone supplementation. Which of the following is a potential side effect? A- Bloating B- Hypotension C- Acne D- Endometrial hyperplasia E- DVT

C-Acne

A 70-year-old male with BPH and an enlarged prostate continues to have urinary symptoms after an adequate trial of tamsulosin. Dutasteride is added to his therapy. In addition to tamsulosin, he is also taking hydrochlorothiazide, testosterone, and vardenafil as needed prior to intercourse. Which of his medications could have an interaction with dutasteride? A. Hydrochlorothiazide. B. Tamsulosin. C. Testosterone. D. Vardenafil.

C. Because dutasteride prevents the conversion of testosterone to the more active form, DHT, these medications have an interaction. Essentially, dutas- teride prevents testosterone from "working." Hydrochlo- rothiazide does not interfere with the metabolism of dutasteride, and dutasteride does not have any effect on the blood pressure-lowering effects of hydrochlorothiazide. Tamsulosin is appropriate in combination with a 5-α reduc- tase inhibitor when the prostate is enlarged. Vardenafil is only prescribed as needed, and the two drugs do not have a pharmacokinetic interaction.

A patient is worried about starting terazosin because he is very sensitive to side effects of medications. Which of the following adverse effects would be most expected in this patient? A. Erectile dysfunction. B. Gynecomastia. C. Dizziness. D. Vomiting.

C. Because of the α-blocking properties, terazosin commonly causes dizziness (this may be related to orthostatic hypotension). ED and gynecomastia would be unexpected with α-blockers. While most any drug may cause nausea and vomiting, terazosin is much more likely to cause dizziness.

When selecting between the available PDE-5 inhib- itors for treatment of ED, which of the following is an important consideration? A. Tadalafil has the shortest half-life of the PDE-5 inhibitors. B. Sildenafil should be given with food to increase absorption. C. Vardenafil ODT doses are not equal to film-coated vardenafil doses. D. Avanafil should be taken at least 1 hour prior to intercourse.

C. The ODT dosage form of vardenafil provides a high systemic concentration of vardenafil, which is higher than that provided by the film-coated tablets. As such, the doses are not interchangeable. Tadalafil has the longest half-life of all PDE-5 inhibitors. Food may delay sildenafil absorption. Avanafil has the quickest onset of action and may be taken 30 minutes prior to intercourse.

A 60-year-old female patient started on a new antihypertensive medication recently. Her blood pressure seems to be under control, but she complains of fatigue, drowsiness, and fainting when she gets up from the bed (orthostatic hypotension). Which of the following drugs is she most likely taking? A. Metoprolol. B. Propranolol. C. Prazosin. D. Clonidine.

C. α-Blockers (prazosin) are more likely to cause orthostatic hypotension compared to β-blockers (metoprolol, propranolol) and α2 agonists (clonidine).

Diethylstilbestrol (DES) should never be used in pregnant women because it is associated with which of the following? A- Deep vein thrombosis B- Feminization of the external genitalia of male offspring C- Infertility and development of vaginal cancer in female offspring D- Miscarriages E- Virilization of the external genitalia of female offspring

C: Diethylstilbestrol (DES) is a nonsteroidal estrogen agonist. Several decades ago, misguided use of the drug in pregnant women appears to have resulted in fetal damage that predisposed female offspring to infertility and a rare form of vaginal cancer. For this reason, the drug should be avoided in pregnant women. Other estrogenic drugs do not appear to have these effects. Although estrogens do increase the risk of deep vein thrombosis, this is not the reason why DES should be avoided.

A 60-year-old man is found to have a prostate lump and an elevated prostate-specific antigen (PSA) blood test. Magnetic resonance imaging suggests several enlarged lymph nodes in the lower abdomen, and an x-ray reveals 2 radiolucent lesions in the bony pelvis. This patient is likely to be treated with which of the following drugs? A- Anastrozole B- Desogestrel C- Leuprolide D- Methyltestosterone E- Oxandrolone

C: Leuprolide is a GnRH agonist used in the treatment of men with prostate cancer. Continuous use leads to downregulation of testosterone production. Initially, the agonist action increases testosterone, causing a tumor flare. To prevent this, flutamide, a competitive antagonist of the androgen receptor, is added until downregulation of testosterone is complete.

Which of the following is contraindicated to use with nitrates? A- Finasteride B- Flutamide C- Leuprolide D- Sildenafil E- Methyltestosterone

D - Sildenafil risk for hypotension

Selective alpha-1 adrenergic antagonists have more effect on the alpha1A receptor subtype found on smooth muscle of prostate and have less effect on blood pressure. Which of the following is a selective alpha-1 adrenergic antagonists for treatment of BPH? A- Doxazosin B- Finasteride C- Prazosin D- Alfuzosin E- Tolterodine

D - Alfuzosin

Flutamide is often given in combination with leuprolide for treatment of prostate cancer. What is the mechanism of action of flutamide? A- Inhibitor of gonadotropin secretion B- Alpha-1 adrenergic antagonists C- 5-alpha-reductase inhibitors D- Antagonist at androgen receptor E- Agonist at androgen receptor

D - Antagonist at androgen receptor

What is the mechanism of action of mifepristone? A- Functions as a synthetic androgen. B- Suppresses LH/FSH by interfering with GnRH C- Functions as a selective progesterone receptor modulator. D- Competitively binds progesterone receptor and antagonizing endometrial and myometrial effects. E- Blocks the alpha-1 adrenergic receptors in the periphery which results in smooth muscle relaxation.

D - Competitively binds progesterone receptors and antagonizes endometrial and myometrial effects.

Which of the following is a potential side effect of progesterone? A- DVT B- Abnormal vaginal bleeding C- Breast discomfort D- Decrease HDL E- Endometrial hyperplasia

D - Decrease HDL

What is the mechanism of action of alfuzosin? Blocks the action of 5-alpha-reductase, the enzyme that converts testosterone to dihydrotestosterone. Inhibits GnRH secretion at the hypothalamus. Inhibits phosphodiesterase type V, thereby increasing levels of cGMP which results in smooth muscle relaxation. Blocks the alpha-1 adrenergic receptors in the periphery which results in smooth muscle relaxation. Antagonist at testosterone receptor.

D- Blocks the alpha-1 adrenergic receptors in the periphery which results in smooth muscle relaxation.

A 26-year old female patient with no significant past medical history presents after trying to get pregnant for over one year. You refer her to a fertility clinic. After a series of labs and imaging, the patient is started on which medication to stimulate ovulation? A- Progesterone B- GnRH antagonist C- Recombinant follicle stimulating hormone D- Clomiphene E- Raloxifene AND what drug class is the answer in?

D- Clomiphene = SERM Raloxifene is not the answer because it used for osteoporosis, receptors on the bone .

You are seeing a patient in follow-up after she was started on hormone replacement therapy 3 months ago. At the time, she had severe menopausal symptoms. She has no significant past medical history. Specifically, she has no prior history of thromboembolic disorder. She has never experienced abnormal vaginal bleeding. She is happy with the symptom relief and asks how long she should continue HRT. A- There is no risk in continuing HRT indefinitely. B- She can transition to estrogen therapy alone in 6 months to minimize side effects. C- Continue therapy for five years then stop to see if symptoms return. D- Continue a 6 to 12-month course, reevaluating at that time to see if continued therapy is necessary. E- You explain that she should not have started hormone replacement therapy in the first place.

D- Continue a 6 to 12-month course, reevaluating at that time to see if continued therapy is necessary.

Which of the following is a progestogen? A- Tamoxifen B- Tamsulosin C- Estrone D- Desogestrel E- Mifepristone

D- Desogestrel

What is the major hormone secreted by the ovary? A- Ethinyl estradiol B- Estrone C- Progesterone D- Estradiol E- Norgestrel

D- Estradiol

Which of the following is used for treatment of primary hypogonadism in male patients? A- Estrone B- Tamsulosin C- Flutamide D- Fluoxymesterone E- Drospirenone

D- Fluoxymesterone

A patient who is taking a PDE-5 inhibitor is diagnosed with angina. Which of the following antianginal medications would be of particular concern in this patient? A. Metoprolol. B. Diltiazem. C. Amlodipine. D. Nitroglycerin.

D. Nitrates, when taken with PDE-5 inhibi- tors, can cause life-threatening hypotension. While metopro- lol, diltiazem, and amlodipine may all lower blood pressure, the interaction with PDE-5 inhibitors is not relevant.

A 70-year-old male needs to be treated with an α-blocker for overflow incontinence due to his enlarged prostate. Which of the following drugs would you suggest in this patent that will not affect his blood pressure significantly? A. Prazosin. B. Doxazosin. C. Phentolamine. D. Tamsulosin. E. Terazosin.

D. Tamsulosin is an α1 antagonist that is more selective to the α1 receptor subtype (α1A) present in the prostate and less selective to the α1 receptor subtype (α1B) present in the blood vessels. Therefore, tamsulosin does not affect blood pressure significantly. Prazosin, doxa- zosin, terazosin, and phentolamine antagonize both these subtypes and cause significant hypotension as a side effect

Which of the following is the BEST description of the mechanism of action of terazosin? A. Terazosin blocks 5-α reductase. B. Terazosin blocks α1A receptors. C. Terazosin blocks PDE-5. D. Terazosin blocks α1A and α1B receptors.

D. Terazosin blocks both the α1A and α1B receptors. Terazosin does not affect 5-α reductase or PDE-5.

Which of the following BEST describes the mechanism of action of alprostadil? A. Alprostadil blocks cGMP. B. Alprostadil blocks nitric oxide. C. Alprostadil increases PDE-5. D. Alprostadil increases cAMP.

D. Through an unknown mechanism, alprostadil (a synthetic prostaglandin) increases levels of cAMP, causing smooth muscle relaxation. Alprostadil does not affect cGMP, nitric oxide, or PDE-5.

A 52-year-old postmenopausal patient has evidence of low bone mineral density. She and her physician are considering therapy with raloxifene or a combination of conjugated estrogens and medroxyprogesterone acetate. Which of the following patient characteristics is most likely to lead them to select raloxifene? A- Previous hysterectomy B- Recurrent vaginitis C- Rheumatoid arthritis D- Strong family history of breast cancer E- Troublesome hot flushes

D: Conjugated estrogens and raloxifene both improve bone mineral density and protect against osteoporosis. The 2 advantages of raloxifene over full estrogen receptor agonists are that raloxifene has antagonist effects in breast tissue and lacks an agonistic effect in endometrium. If a patient's uterus was removed by surgery, the difference in the endometrial effect is moot. In patients with a strong family history of breast cancer, raloxifene may be a better choice than a full estrogen agonist because it will not further increase the woman's risk of breast cancer and may even lower her risk.

A 23-year-old woman desires a combined oral contraceptive for pregnancy protection. Which of the following patient factors would lead a health professional to recommend an alternative form of contraception? A- Evidence of hirsutism B- History of gastroesophageal reflux disease and is currently taking omeprazole C- History of pelvic inflammatory disease D- History of migraine headache that is well controlled by sumatriptan E- She plans to use this contraceptive for about 1 year and will then attempt to become pregnant

D: Estrogen-containing hormonal contraceptives increase the risk of episodes of migraine headache.

A teenager seeks postcoital contraception. Which of the following preparations will be effective for this purpose? A- Clomiphene B- Tamoxifen C- Diethylstilbestrol (DES) D- Mifepristone

D:Mifepristone, an antagonist at progesterone and glucocorticoid receptors, has a luteolytic effect and is effective as a postcoital contraceptive. When combined with a prostaglandin, it is also an effective abortifacient.

Ms. Murphy is a 39 year old female presenting to the office requesting B.C. What are your thoughts on how to administer OCP Vitals: Temp 36.6 HR 77 RR 18 BP 160/95 O2 sat 100%

Does she smoke, her age, hx of DVT, hx of PE, cardiac disease, breast CA, abnormal vaginal bleeding, impaired liver fxn, migraine hx - Shared decision making

Ms. Smith is a 22 year old female presenting to the office requesting birth control. What are you thoughts on how to administer OCP Vitals: Temp 36.6 HR 62 RR 20 BP 120/65 O2 sat 100

Does she smoke, her age, hx of DVT, hx of PE, cardiac disease, breast CA, abnormal vaginal bleeding, impaired liver fxn, migraine hx - Shared decision making

Which of the following is the treatment of choice for atrophic vaginitis in perimenopausal women? A- Oral progesterone B- Tamoxifen C- Oral estrogen D- Topical progesterone E- Topical estrogen

E - topical estrogen

Finasteride has efficacy in the prevention of male-pattern baldness by virtue of its ability to do which of the following? A- Competitively antagonize androgen receptors B- Decrease the release of gonadotropins C- Increase the serum concentration of sex hormone-binding globulin D- Inhibit the synthesis of testosterone E-Reduce the production of dihydrotestosterone

E: Finasteride inhibits 5α-reductase, the enzyme that converts testosterone to DHT, the principal androgen in androgen-sensitive hair follicles.

A 50-year-old woman with a positive mammogram undergoes lumpectomy and a small carcinoma is removed. Biochemical analysis of the cancer reveals the presence of estrogen and progesterone receptors. After this procedure, she will probably receive which of the following drugs? A- Danazol B- Flutamide C- Leuprolide D- Mifepristone E- Tamoxifen

E: Tamoxifen has proved useful in adjunctive therapy of breast cancer; the drug decreases the rate of recurrence of cancer.

A 25-year old female patient presents 4 days after unprotected intercourse requesting emergency contraception. Which of the following are treatment options you can discuss with her? A- Mifepristone and Copper IUD B- Copper IUD and Levonorgestrel C- Levonorgestrel and Ulipristal D- Ulipristal and Copper IUD E- Mifepristone and Levonorgestrel

NO Levonorgestrel --> outside the window to take it D- Ulipristal and Copper IUD


Conjuntos de estudio relacionados

Bed Positions / Pictures & Bed positions

View Set

mastering biology chapter 4 questions

View Set

Chapter 5: Externalities, Enviornmental Policy, and Public Goods

View Set

Week 4- Statistics Chapter 4 Review Quiz

View Set

A & P Ch.12 Nervous System III Senses

View Set